Você está na página 1de 40

CLAT

Law Prep Mock Test Series

Duration: 120 Minutes Max. Marks : 150 Test Code: LPMTS-013-10020

Student Name :

Batch : Enrollment No. :

INSTRUCTIONS TO CANDIDATES
1. No Clarification on the question paper can be sought. Answer the questions as they are.
2. There are 150 multiple choice objective type questions. Answer ALL the questions
3. Each question carries ONE mark. Total marks are 150.

English 30 30
Logical Reasoning 30 30
Current Affairs 40 40
Legal Reasoning 35 35
Quantitative Technique 15 15
150 150

4. There will be Negative Marking. 0.25 marks will be deducted for every wrong answer.
5. Candidates have to indicate the correct answer by darkening one of the four responses provided, with BLACK/BLUE BALL
POINT PEN in the OMR Answer Sheet.
Example: For the question, "The Emergency Provisions of Indian Constitution have been borrowed from.", the correct
answer is (B).
The candidate has to darken the corresponding circle as indicated below:
(a) Japan (b) Germany (c) USSR (d) U.S.A.

Right Method Wrong Methods


a b c d a b c d a b c d a b c d

6. Answering the question by any method other than the method indicated above shall be considered wrong answer.
7. More than one response to a question shall be counted as wrong answer.
8. The candidates shall not write anything on the OMR Answer Sheet other than the details required and in the spaces
provided for.
9. After the Test is over, the candidate has to return the test booklet along with the OMR Answer Sheet to the invigilator.
10. The use of any unfair means by any candidate shall result in the cancellation of his/her candidature.
11. Impersonation is an offence and the candidate, apart from disqualification, may have to face criminal prosecution.
12. Electronic gadgets like mobile phones, pagers or calculators etc. are strictly not permitted inside the Test Centre / Hall.
13. The candidates shall not leave the hall before the Test is over.

H.O. : B-61, St. Paul School Road, Shastri Nagar, JODHPUR - 342 003
Ph. 0291-2655777, 94141 43101, 76659 44999
www.lawpreptutorial.com
ALL

AIR1
INDIA Saumya
TOPPERS CLAT 2019
AILET 2019

3 4
AIR
1
AIR
AIR
Harsh Gunjan Rajat
AILET 2019 CLAT 2018 CLAT 2017

w w w. l a w p r e p t u t o r i a l . c o m
Section I: English

PASSAGE - 1

Directions for Questions 1 to 5: A passage is given with the following questions. Read the passage
carefully and choose the best answer to each question out of four alternatives.
A nationwide extension of NRC is bizarre, and a repeat of it in Assam illogical
Home Minister Amit Shah's announcement of a proposal for a nationwide National Register of Citizens
(NRC) is worrisome on several counts, not the least of which is the apparent inability to learn from the
experience of carrying out the humongous exercise in Assam. The government, he said, would also
reintroduce the Citizenship Amendment Bill (CAB) in Parliament that envisages the grant of Indian
citizenship to all refugees from minority communities in Bangladesh, Pakistan and Afghanistan. In all three
nations Muslims are in a majority, and therefore, the Bill effectively denies benefit to Muslim minorities from
other neighbouring countries, including Myanmar where Rohingya Muslims face persecution. Along with the
promised combination of the NRC and CAB, the Home Minister announced that the NRC process would
"naturally" be conducted in Assam again with the rest of the country. Interestingly, this comes just days
after Ranjan Gogoi, who supervised the NRC process, demitted office as Chief Justice of India. Clearly, the
Assam proposal will be in defiance of the Supreme Court, which directed the entire NRC registration
specific to Assam through all its tortuous details. There is still no clarity on what the end results mean for
the 19 lakh plus people who find themselves outside the NRC, potentially stateless and a risk of
"deportation" to Bangladesh, which refuses to acknowledge, let alone accept, them. Given that the NRC
process in Assam was rooted in the specificities of the 1985 Assam Accord, and as the government never
tires of saying, a court-mandated process, extending it to the entire country is both illogical and bizarre.
Flawed it might have been, but the NRC exercise, overseen by the Supreme Court, involved the active
participation of the Central and State governments. For the government to repeat the exercise merely
because the numbers thrown up are politically inconvenient for the ruling BJP, makes no sense at all. If
there is a lesson from Assam, it is that there is no right way of going through a process such as the NRC.
Like the CAB, which pointedly discriminates against Muslims, and is loaded against the right to equality
and equal protection before the law as enshrined in Article 14 of the Constitution, there are genuine fears
that a nationwide NRC will target Muslims. Details of how such an exercise will be carried out are, of
course, not yet known. In the case of Assam, there was a cut-off date March 25, 1971 after which all
foreigners as per the Assam Accord were to be "detected, deleted and expelled in accordance with law".
Presumably, the centre will come out with a cut-off for the nationwide NRC, but it will be an arbitrary one.
Given the dangers that lurk within such exercises, the government would do well to abandon the nationwide
NRC-CAB combination. Indians can certainly be spared this pain.
1. In which state of India, the NRC is being implemented for the first time?
(a) Arunachal Pradesh (b) Assam (c) West Bengal (d) Tripura

2. According to the data, how many people are deprived of NRC in Assam?
(a) More than 19 lakh (b) More than 20 lakh (c) More than 25 lakh (d) More than 10 lakh

3. What is the cut-off year to determine the foreigners in Assam, who are deprived of NRC?
(a) 1972 (b) 1971 (c) 1985 (d) 1977

4. What is the antonym of the underlined word in the passage?


(a) Expulsion (b) Recruitment (c) Eviction (d) Removal

5. Under which Article of the Constitution of India 'the right to equality and equal protection before the law' is
enshrined?
(a) Article 11 (b) Article 12 (c) Article 15 (d) Article 14

03
PASSAGE - 2

Directions for Questions 6 to 10: A passage is given with the following questions. Read the passage
carefully and choose the best answer to each question out of four alternatives.

On the Ayodhya dispute, mediation is the only way to deal with faith-related arguments

It has taken 70 years for the Babri Masjid-Ram Janmabhoomi litigation to come close to finality. The
appeals against the Allahabad High Court's judgment on the title suits filed by both Hindu and Muslim
parties have been heard by a five judge Bench of the Supreme Court for 40 days. The court has reserved
judgment after hearing impassioned, sometimes acrimonious, arguments. The embers of competing
claims, which remained alive well into the 20th century, were stoked by the surreptitious installation of Lord
Ram's idol on the night of December 22/23, 1949 under the structure's central dome. Suits were filed over
the years by both sides. It was not until the 1980s that the Ayodhya dispute was used for political
mobilesation by Hindu nationalist groups. After a court ordered the reopening of the structure's doors in
1986, the Bharatiya Janata Party saw the scope for a national movement that would one day catapult it to
power. With the VHP and Bajrang Dal launching a movement for the 'retrieval' of the site for the
construction of a grand Ram Mandir, a dispute over title and the right of worship transmogrified into an
intractable litigation predicated on faith. It is possible that the case's emotive nature and its potential for
dividing society prevented its early disposal. The matter was ultimately disposed of by the High Court
Bench in 2010. The decision - a three-way division of the disputed area among the deity, the Nirmohi
Akhara and the Muslim side - satisfied no one and the matter went up to the Supreme Court.
As the final verdict is awaited, it cannot be forgotten that the demolition of the disputed structure in
December 1992 was an egregious crime against the country's secular fabric and its constitutional ethos.
The purported evidence of a Hindu structure beneath the mosque came up only in excavations made after
the structure was razed. Any decision made on such evidence, which would not have been available to the
court if the suits had been disposed of in earlier decades, might amount to the judicial system legitimising
the demolition. Even otherwise, the fact that a modern democracy should have been saddled with litigation
motivated by historical revanchism is execrable in itself There can be no judicial standards to settle a faith-
based argument.
There is some talk of a "settlement" based on mediation efforts at the court's behest. A mediated
settlement would be welcome, even though it is not clear if all sides are on board. However, if the outcome
is not to be based purely on the rule of law, it would be better there is a mediated settlement.

6. Which group launched a movement for the retrieval of the site to construct a grand Ram Mandir in
Ayodhya?
(a) BJP (b) RSS (c) VHP and Bajrang Dal (d) Bajrang Dal

7. Which evidence found beneath the mosque came up in excavation in disputed land?
(a) Evidence of Hindu Structure (b) Evidence of mosque
(c) Evidence of demolition of another structure (d) None of the above

8. When did court order the reopening of the doors of the disputed structure?
(a) 1986 (b) 1992 (c) 2010 (d) 1949

9. An application of Ayodhya-Babri Masjid case was filed in the Supreme Court against the decision of-
(a) Allahabad High Court (b) Delhi High Court
(c) Patna High Court (d) None of the above

10. What is the synonym of the underlined word in the passage?


(a) Construction (b) Renaissance (c) Rehabilitation (d) Destruction

04
PASSAGE – 3

Directions for Questions 11 to 14: A passage is given with the following questions. Read the passage
carefully and choose the best answer to each question out of four alternatives.
The Modi-Xi summit might restore the bonhomie in ties that saw strains recently
A unique relationship requires unique and imaginative ways to manage differences. It is for this reason
that India and China, with a 3,380 km common boundary, thousands of years of a shared history, and half -
a-century of boundary disputes and tensions has always needed unique mechanisms. The second
"informal summit" between Prime Minister Narendra Modi and China's President Xi Jinping that begins on
Friday is one such way for the two countries to deal with the ebb and flow in ties. The leaders last held an
informal summit in Wuhan, in the wake of the Doklam crisis, when the time they spent visibly improved the
atmospherics around the relationship. In the months prior to and following the Wuhan summit, the two
nations brought down tensions along the boundary, initiated a new dialogue on trade, which led to more
market access and a small indent in the $53-billion trade deficit between them, and saw more international
coordination including at the WTO, and on climate change and terrorism. Earlier this year, China joined
UNSC members in a statement condemning the Pulwama attack and then in May reversed its decade-old
position by allowing the UNSC listing of the Jaish e Mohammad chief Masood Azhar, Since the August 5
decision by the government on Article 370 and the bifurcation of Jammu and Kashmir, and Ladakh, and
the strong reaction from China, these positive steps seemed to have been stalled. If Beijing's decision to
raise the Indian government's move at the UNSC was a clear break from the "Wuhan Spirit", its
subsequent statements including at the UN General Assembly have alienated New Delhi further. Optics
closer to the summit have been more troublesome, and Mr. Xi's invitation to the Pakistan Prime Minister
Imran Khan to visit Beijing in the same week that Mr. Xi meets Mr. Modi is clearly a negative signal ahead
of the Chennai summit. India's decision to hold mountain combat exercises in Arunachal Pradesh just prior
to the summit is another in the series of red flags raised, which had even cast a doubt over whether the
summit would go ahead.
A, a result, Mr. Modi and Mr. Xi have their task cut out for them: in restoring some of the bonhomie
from last year, while charting a course for ties ahead. In the immediate future, it is hoped that Mr. Xi will
assuage India's concerns on trade issues with a view to meeting the deadline for the ASEAN-led RCEP
free trade agreement in November, and possibly direct special representatives on the boundary issue to
speed up their talks. For the larger picture, it is important that they end the downslide in bilateral ties and
set up more robust communication in order to address each other's concerns in a timely manner. If the
Wuhan summit focused on a reset between India and China, the Chennai summit will be successful if it
ensures that there is no rollback m the relationship, a goal that will no doubt be aided by the
Mamallapuram setting, which will remind the leaders of the ancient and traditional ties between the two
countries based on trade, travel and faith.
11. What is a negative signal ahead of Chennai Summit?
(a) China reversed its decade-old position by allowing the UNSC listing of the Jaish Mohammad chief,
Masood Azhar
(b) Article 370 and the bifurcation of Jammu and Kashmir, and Ladakh
(c) China's invitation to the Pakistan Prime Minister lmran Khan to visit Beijing
(d) Boundary disputes between India and China

12. Why was informal summit between PM Narendra Modi and China's President Xi Jinping held in Wuhan?
(a) For a dialogue on trade (b) Climate change
(c) Terrorism (d) To bring down tensions along with boundary

13. How much boundary does India share with China?


(a) 3,370km (b) 3,380km (c) 3,890km (d) 3,390km

14. What is the antonym of the underlined word in the passage?


(a) Enmity (b) Amity (c) Friendliness (d) Affability

05
Direction for Question 15: Each of the following questions has a paragraph from which the last sentence
has been deleted. From the given options, choose the one that completes the paragraph in the most
appropriate way.
15. The government of India allows corruption. How else could it have become the scourge that it has today?
The Prime Minister may be thinking of removing a happy clause in the law, happy for the thieves that is. A
clause supposedly put there to protect public servants from wrongful harassment which blatantly helps
crooks. Government departments and ministries have been misusing a constitutional provision (Article 311)
in which the CVC - Central Vigilance Commission - has to seek prosecution sanction from the government
before beginning a formal probe against allegedly corrupt officials.___________.
(a) The clause has opened a floodgate for the corrupt with scores of officials escaping prosecution
because of it.
(b) No one wants seems to have the will to get rid of it, at least not our elected representatives.
(c) The clause is a seed planted by our law makers during the drafting of the anti-corruption act.
(d) In other words the government has laid out a red carpet for the thieves.

PASSAGE - 4
Directions for Questions 16 to 18: A passage is given with the following questions. Read the passage
carefully and choose the best answer to each question out of four alternatives.
The Thirty-None Steps – by John Buchan
I did not give him very close attention. The fact is, I was more interested in his own adventures than in
his high politics. I reckoned that Karolides and his affairs were not my business, leaving all that to him. So
a lot that he said slipped clean out of my memory. I remember that he was very clear that the danger to
Karolides would not begin till he had got to London, and would come from the very highest quarters, where
there would be no thought of suspicion. He mentioned the name of a woman–Julia Czechenyi–as having
something to do with the danger. She would be the decoy, I gathered, to get Karolides out of the care of
his guards. He talked, too, about a Black Stone and a man that lisped in his speech, and he described very
particularly somebody that he never referred to without a shudder–an old man with a young voice who
could hood his eyes like a hawk.
He spoke a good deal about death, too. He was mortally anxious about winning through with his job,
but he didn’t care a rush for his life. ‘I reckon it’s like going to sleep when you are pretty well tired out, and
waking to find a summer day with the scent of hay coming in at the window. I used to thank God for such
mornings way back in the Blue-Grass country, and I guess I’ll thank Him when I wake up on the other side
of Jordan.’
Next day he was much more cheerful, and read the life of Stonewall Jackson much of the time. I went
out to dinner with a mining engineer I had got to see on business, and came back about half-past ten in
time for our game of chess before turning in.

16. What does the following line analogize? “I reckon it’s like going to sleep when you are pretty well tired out,
and waking to find a summer day with the scent of hay coming in at the window.”
(a) What the narrator imagines the experience of death to be like.
(b) The sensation of “winning through” with one’s job.”
(c) The speaker is describing his fluctuating mood as the danger to Karolides approaches.
(d) What the narrator’s companion imagines the experience of death to be like.

17. It can be gathered from the passage that Karolides:


(a) Is a woman (b) Is the narrator’s companion
(c) Is presently in danger (d) Will be threatened by surreptitious forces

06
18. The narrator’s greater interest in his companion’s “adventures than in his high politics” suggests that:
(a) The narrator is not a political man.
(b) The narrator is indifferent to his companion.
(c) The narrator is a man of action.
(d) More can be learned from the companion’s description of events than his personal beliefs for
committing to a cause.

Directions for Questions 19 to 23: Read the given passage and answer the following questions based on
the passage.
There has been much development recently with respect to the Protection of Children from Sexual
Offences (POCSO) Act, 2012. With an objective of stopping the rampant sexual abuse of children, the
Protection of Children from Sexual Offences (Amendment) Bill, 2019 was introduced in the Rajya Sabha in
July, and later passed by both Houses of Parliament. It is all set to become the ‘law of the land’. The
present bill is welcome in certain respects as it specifically defines what ‘child pornography’ is; ‘using a
child for pornographic purposes’ and for ‘possessing or storing pornography involving a child’ is punishable.
It has also widened the ambit of ‘Aggravated sexual assault’.
The highlight of the Bill is the introduction of the death penalty for the rape of minors. The Bill, in its
object clause, justifies this by referring to the judgments of the Supreme Court in Machhi Singh (1983) and
Devender Pal Singh (2002) in which the court has held that the death penalty can be awarded only in rarest
of rare cases. Thus the intention of the Bill is to have a deterrent effect; but it can be argued that the
introduction of the death penalty may backfire in cases of child sexual abuse and even have a catastrophic
effect. Often, the perpetrators of abuse are family members and having such penalty in the statute book
may discourage the registration of the crime itself. Also, it may threaten the life of the minor as the
maximum punishment for murder is also the death sentence.

The Justice J.S. Verma Committee, which was constituted in 2013 in the aftermath of the Nirbhaya
case, after due deliberations found itself against the imposition of death penalty in rape cases. The 262nd
Report of the Law Commission of India, 2015, also provides for abolition of the death penalty except in
terror cases.
Today, the death penalty has become a prominent tool of symbolic legislation — a political statement
indeed. Many a time, the Government, by introducing the death penalty, portrays itself to be strict and
serious with regard to such offences. It largely diverts attention from the core issues of infrastructural
apathy, procedural lapses and trial delays and conveniently evades the fact that ‘it is the certainty of
punishment rather than its severity which has deterrence in real sense’. It is pertinent to note here that even
a year-and-a-half after the passage of the Criminal Law (Amendment) Bill, 2018, which introduced the
death penalty for rape of a minor girl, such incidents have not been under check. The debate here is not
about retaining or abolishing the death penalty but the probable _______(A) of its provision in the Act.
The Supreme Court has recently taken cognisance of the sexual abuse of children, directing its registry
to file a case as writ petition with cause title “In-re Alarming Rise in The Number of Reported Child Rape
Incidents”. The court has also observed that it intends having a ‘zero tolerance policy’ toward child rape. As
data on sexual crimes against children collected by the court show, 24,212 FIRs were filed across India
from January to June this year. According to National Crime Records Bureau data of 2016, the conviction
rate in POCSO cases is 29.6% while pendency is as high as 89%. The prescribed time period of two
months for trial in such cases is hardly complied with.
The court has also taken note of the delay in trials, in turn directing the Central Government to set up
special courts within 60 days of the order in each district having more than 100 pending cases under the
Act. It is to be seen how long it takes to comply with the order.
The Criminal Law (Amendment) Act, 2018 introduced the death penalty for rape of girls below the age
of 12. At the same time, the POCSO Act, under Section 42, provides that where the same act constitutes
an offence under the said Act and any other law, then the offender will be punished under the Act or such

07
law, whichever provides for greater punishment. This has created an issue as the effect of such an
amendment was death penalty for rape of minor girls but not for assault against minor boys.
The proposed Bill does away with such a discrepancy. It is gender neutral and provides for the death
penalty for “aggravated penetrative sexual assault of a child”, thus bringing both these pieces of legislation
on a par with each other in this respect. With these amendments and with the Supreme Court considering
child abuse “intolerable”, there seems to be reasonable hope now that vulnerable children could be safer.
The Bill is a step forward in preventing child abuse but the consequences of providing for the death penalty
need to be closely observed.
19. What could be the most appropriate title of this passage?
(a) Pondering over POSCO Bill
(b) Life sentence under POSCO Bill
(c) Diverting attention from core issues under POSCO Bill
(d) All of the above

20. In what aspects, the present Protection of Children from Sexual Offence (amendment) Bill, 2019 is
welcomed?
(a) It defines child pornography specifically
(b) It has broadened the domain of aggravated sexual assault
(c) Death penalty for rape of minors is included in this Bill.
(d) All of the above

21. Why it is said in the italicized sentence, ‘Today, the death penalty has become a prominent tool of symbolic
legislation’ mean according to the above passage?
(a) The idea behind death penalty is to portray strict and serious attitude towards such offences
(b) It deviates the attention of public from main issues such as infrastructural indifference, lapse in
procedural hearings and delayed trials.
(c) It vanishes the fact that death penalty is a certain punishment which otherwise is constraining in reality.
(d) All of the above

22. Which one of the following statements is not true about POSCO Bill according to the above paragraph?
(a) Protection of Children from Sexual Offences Amendment Bill, 2019 was introduced in July in Rajya
Sabha.
(b) Introduction of Death penalty may fall back in case of child abuse.
(c) The court has held the death penalty for rarest of rare cases.
(d) It intends to have a zero tolerance policy towards child rape cases

23. Choose among the following most suitable word to fill the blank given in (A)
(a) Acumen (b) Multifarious (c) Ramifications (d) Exacerbate

Directions for Questions 24 to 28: A passage is given with the following questions. Read the passage
carefully and choose the best answer to each question out of four alternatives.
With the CJI under the RTI Act, there will be greater transparency by public authorities
The adage, "sunlight is the best disinfectant" is often used to delineate the need for disclosure of
matters related to public interest through the Right to information mechanism. The declaration of assets by
minister and legislators, besides electoral candidates, has gone a long way in shedding light on public
authorities and provided the citizenry more relevant information about their representatives. Yet, judges of
the Supreme Court had hitherto refused to share information on their personal assets, citing the express
lack of public interest. The welcome ruling by a five-member Constitution Bench of the Supreme Court that
the office of the Chief Justice of India is a "public authority" under the RTI Act, as much as the apex court
itself, now enables the disclosure of information such as the judges, personal assets. The judgement’s

08
majority opinion, written by Justice Sanjiv Khanna, emphasized the need for transparency and
accountability and that “disclosure is a facet of public interest”. In concurring opinions, Justice D. Y.
Chandrachud asserted that judicial independence was not secured by secrecy while Justice N. V. Ramana
argued for the need of a proper calibration of transparency in light of the importance of judicial
independence. The Bench unanimously argued that the right to know under the RTI Act was not absolute
and this had to be balanced with the right of privacy of judges. But the key take away from the judgement is
that disclosure of details of serving judges’ personal assets was not a violation of their right to privacy. The
main opinion also argued that information related to issues such as judicial appointments will also be
subject to the test of public interest and procedures mandated in the RTI Act that specify that views of third
parties (in this case, judges) must be sought. While laying out the importance of the assessment of public
interest in any RTI query besides bringing the office of the CJI under the purview of the Act, the decision
has gone on to uphold the Delhi High Court verdict in 2010. The RTI Act is a strong weapon that enhances
accountability, citizen activism and, consequently, participative democracy, even if its implementation has
come under strain in recent years due mainly to the Central government’s apathy and disregard for the nuts
and bolts of the Act. Yet, despite this, the Supreme Court judgement paves the way for greater
transparency and could now impinge upon issues such as disclosure, under the RTI Act, by other
institutions such as registered political parties. This is vital as political party financing is a murky area today,
marked by opacity and exacerbated by the issue of electoral bonds, precluding citizens from being fully
informed on sources of party incomes.
24. What decision did the Supreme Court give in the matter of registering RTI regarding the post of its CJI?
(a) The SC considered the post of CJI under the RTI.
(b) SC refused to accept the CJI office under the RTI.
(c) SC refused to accept the CJI office as a public authority.
(d) None of the above.

25. Why was the CJI post of SC brought under RTI?


(a) For transparency and accountability (b) Because it is good for public interest
(c) Both (a) and (b) (d) None of the above

26. Why the transparency is must in SC's CJI office?


(a) Due to independence of SC (b) To maintain the dignity of SC
(c) To maintain the impartiality of SC (d) All of the above

27. What was the main key feature of this case in SC?
(a) Disclosure of details of serving judges, assets.
(b) To bring more transparency in judicial decrees.
(c) Clear disclosure of judicial appointments.
(d) None of the above

28. What is the antonym of the underlined word in the passage?


(a) Formerly (b) Long ago (c) Currently (d) Once

Directions for Questions 29 & 30: Four alternative summaries are given below the text. Choose the
option that best captures the essence of the text.
29. When the next full-scale global financial crisis hits, let it not be said that the International Monetary Fund
never took a stab at, forestalling it. Recently, the IMF proposed a new global tax on financial institutions
loosely in proportion to their size, as well as a tax on banks’ profits and bonuses.
(a) The IMF has proposed a new global tax on financial institutions and banks in an attempt to forestall
future financial crises.
(b) The IMF’s new global tax on financial institutions and banks will prevent future financial crises.
(c) The IMF has proposed a new global tax on financial institutions and banks which will forestall financial
crises.

09
(d) The IMF has proposed a tax on financial institutions’ and banks’ profits’ and bonuses that will prevent
financial crises.

30. Russia and the United States have signed a new strategic nuclear-arms reduction treaty. Officially, the
treaty cuts their weapons by one-third; in fact, each party will decommission only several dozen.
Nevertheless, the treaty is a considerable achievement. It normalizes political relations between the two
countries, thereby facilitating their further cooperation and rapprochement.
(a) The new strategic arms reduction treaty signed between Russia and the United States may facilitate
further cooperation between them.
(b) The new strategic arms reduction treaty signed between Russia and the United States may lead to
cordial relations between them.
(c) Russia and the United States have signed a strategic nuclear-arms reduction treaty to reduce weapons
by one-third.
(d) Russia and the United States have signed a new strategic nuclear-arms reduction treaty which
normalizes political relations between the two countries.

CLAT 2020
New CLAT
PATTERN

N E T E S T S E R I E S
ONL I ` 2500/-
• 20 ONLINE MOCK TEST ENROLL
• 5 OFFLINE MOCK TEST
• CURRENT GK MAGAZINE (ONLINE)
NOW
www. lawpreptutorial.com

• ONLINE TOPIC TEST


ENQUIRY HELPLINE
076659 44999

10
Section II: Logical Reasoning

Directions for Questions: Each of the critical reasoning questions is based on a short argument, a set of
statements, or a plan of action. For each question, select the best answer of the choices given.
31. Beginning in 1966 all new cars sold in Morodia were required to have safety belts and power steering.
Previously, most cars in Morodia were without these features. Safety belts help to prevent injuries in
collisions, and power steering helps to avoid collisions in the first place. But even though in 1966 one-
seventh of the cars in Morodia were replaced with new cars, the number of car collisions and collision-
related injuries did not decline.
Which of the following, if true about Morodia, most helps to explain why the number of collisions and
collision-related injuries in Morodia failed to decline in 1966?
(a) Because of a driver-education campaign, most drivers and passengers in cars that did have safety
belts used them in 1966.
(b) Most of the new cars bought in 1966 were bought in the months of January and February.
(c) In 1965, substantially more than one-seventh of the cars in Morodia were replaced with new cars.
(d) An excessive reliance on the new safety features led many owners of new cars to drive less cautiously
in 1966 than before.

32. Enterprise Bank currently requires customers with checking accounts to maintain a minimum balance or
pay a monthly fee. Enterprise plans to offer accounts with no monthly fee and no minimum-balance
requirement; to cover their projected administrative costs of $3 per account per month they plan to charge
$30 for overdrawing an account. Since each month on average slightly more than 10 percent of
Enterprise’s customers overdraw their accounts, bank officials predict the new accounts will generate a
profit.
Which of the following, if true, most strongly supports the bank officials’ prediction?
(a) Some of Enterprise Bank’s current checking account customers are expected to switch to the new
accounts once they are offered.
(b) One third of Enterprise Bank’s revenues are currently derived from monthly fees tied to checking
accounts.
(c) Many checking account customers who occasionally pay a fee for not maintaining a minimum balance
in their account generally maintain a balance well above the minimum.’
(d) Customers whose checking accounts do not have a minimum-balance requirement are more likely than
others to overdraw their checking accounts.

33. In virtually any industry, technological improvements increase labor productivity, which is the output of
goods and services per person-hour worked. In Parland’s industries, labor productivity is significantly higher
than it is in Vergia’s industries. Clearly, therefore, Parland’s industries must, on the whole, be further
advanced technologically than Vergia’s are.
The argument is most vulnerable to which of the following criticisms?
(a) It offers a conclusion that is no more than a paraphrase of one of the pieces of information provided in
its support.
(b) It presents as evidence in support of a claim information that is inconsistent with other evidence
presented in support of the same claim.
(c) It takes one possible cause of a condition to be the actual cause of that condition without considering
any other possible causes.
(d) It takes a condition to be the effect of something that happened only after the condition already existed.

34. Chaco Canyon, a settlement of the ancient Anasazi culture in North America, had massive buildings. It
must have been a major Anasazi center. Analysis of wood samples shows that some of the timber for the
buildings came from the Chuska and San Mateo mountains, 50 miles from Chaco Canyon. Only a major
cultural center would have the organizational power to import timber from 50 miles away.

11
In the argument given, the two portions in boldface play which of the following roles?
(a) The first is a premise used to support the argument’s main conclusion; the second is the argument’s
main conclusion.
(b) The first is the argument’s main conclusion; the second is a premise used to support the conclusion.
(c) The first is one of two premises used to support the argument’s main conclusion; the second is the other
of those two premises.
(d) The first is a premise used to support the argument’s main conclusion; the second is a premise used to
support another conclusion drawn in the argument.

35. The Maxilux car company’s design for its new luxury model, the Max 100, included a special design for the
tires that was intended to complement the model’s image. The winning bid for supplying these tires was
submitted by Rubco. Analysts concluded that the bid would only just cover Rubco’s costs on the tires, but
Rubco executives claim that winning the bid will actually make a profit for the company.
Which of the following, if true, most strongly justifies the claim made by Rubco’s executives?
(a) In any Maxilux model, the spare tire is exactly the same make and model as the tires that are mounted
on the wheels.
(b) Rubco holds exclusive contracts to supply Maxilux with the tires for a number of other models made by
Maxilux.
(c) The production facilities for the Max 100 and those for the tires to be supplied by Rubco are located
very near each other.
(d) When people who have purchased a carefully designed luxury automobile need to replace a worn part
of it, they almost invariable replace it with a part of exactly the same make and type.

36. Which of the following most logically completes the passage?


Most bicycle helmets provide good protection for the top and back of the head, but little or no protection for
the temple regions on the sides of the head. A study of head injuries resulting from bicycle accidents
showed that a large proportion were caused by blows to the temple area. Therefore, if bicycle helmets
protected this area, the risk of serious head injury in bicycle accidents would be greatly reduced, especially
since ___________.
(a) among the bicyclists included in the study’s sample of head injuries, only a very small proportion had
been wearing a helmet at the time of their accident
(b) even those bicyclists who regularly were helmets have a poor understanding of the degree and kind of
protection that helmets afford
(c) a helmet that included protection for the temples would have to be somewhat larger and heavier than
current helmets
(d) the bone in the temple area is relatively thin and impacts in that area are thus very likely to cause brain
injury

37. Which of the following most logically completes the argument?


In a typical year, Innovair’s airplanes are involved in 35 collisions while parked or being towed in airports,
with a resulting yearly cost of $1,000,000 for repairs.
To reduce the frequency of ground collisions, Innovair will begin giving its ground crews additional training,
at an annual cost of $500,000. Although this will cut the number of ground collisions by about half at best,
the drop in repair costs can be expected to be much greater, since___________.
(a) most ground collisions happen when ground crews are rushing to minimize the time a delayed airplane
spends on the ground
(b) a ground collision typically occurs when there are no passengers on the airplane
(c) the additional training will focus on helping ground crews avoid those kinds of ground collisions that
cause the most costly damage
(d) the $500,000 cost figure for the additional training of ground crews includes the wages that those crews
will earn during the time spent in actual training

12
38. Many agriculturally intensive areas of the world are beginning to encounter water scarcity problems. As a
result, many farmers in these areas are likely to reduce their output as the water supply they need in order
to maintain production shrinks. However, one group of farmers in such a region plans to increase their
production by implementing techniques for water conservation.
Which of the following, if true, would most strongly support the prediction that the group’s plan will
succeed?
(a) Farmers that can gain a larger share of the food market in their regions will be better positioned to
control more water resources.
(b) Most agricultural practices in areas with water shortages are water-intensive.
(c) Other regions of the world not facing water shortages are likely to make up for the reduction in
agricultural output.
(d) More than half the water used for agriculture in the farmers’ region is lost to evaporation or leakage
form irrigation channels.

39. Hollywood restaurant is replacing some of its standard tables with tall tables and stools. The restaurant
already fills every available seat during its operating hours, and the change in seating arrangements will not
result in an increase in the restaurant’s seating capacity. Nonetheless, the restaurant’s management
expects revenue to increase as a result of the seating change without any concurrent change in menu,
prices, or operating hours.
Which of the following, if true, provides the best reason for the expectation?
(a) One of the taller tables takes up less floor space than one of the standard tables.
(b) Diners seated on stools typically do not linger over dinner as long as diners seated at standard tables.
(c) Since the restaurant will replace only some of its standard tables, it can continue to accommodate
customers who do not care for the taller tables.
(d) Few diners are likely to avoid the restaurant because of the new seating arrangement.

40. A major network news organization experienced a drop in viewership in the week following the airing of a
controversial report on the economy. The network also received a very large number of complaints
regarding the report. The network, however, maintains that negative reactions to the report had nothing to
do with its loss of viewers.
Which of the following, if true, most strongly supports the network’s position?
(a) The other major network news organizations reported similar reductions in viewership during the same
week.
(b) The viewers who registered complaints with the network were regular viewers of the news
organization’s programs.
(c) Major network news organizations publicly attribute drops in viewership to their own reports only when
they receive complaints about those reports.
(d) This was not the first time that this network news organization has aired a controversial report on the
economy that has inspired viewers to complain to the network.

41. Only a reduction of 10 percent in the number of scheduled flights using Greentown’s airport will allow the
delays that are so common there to be avoided. Hevelia airstrip, 40 miles away, would, if upgraded and
expanded, be an attractive alternative for fully 20 percent of the passengers using Greentown airport.
Nevertheless, experts reject the claim that turning Hevelia into a full-service airport would end the chronic
delays at Greentown.
Which of the following, if true, most helps to justify the experts’ position?
(a) Turning Hevelia into a full-service airport would require not only substantial construction at the airport
itself, but also the construction of new access highways.
(b) A second largely undeveloped airstrip close to Greentown airport would be a more attractive alternative
than Hevelia for many passengers who now use Greentown.
(c) Hevelia airstrip lies in a relatively undeveloped area but would, if it became a full-service airport, be a
magnet for commercial and residential development.

13
(d) Several airlines use Greentown as a regional hub, so that most flights landing at Greentown have many
passengers who then take different flights to reach their final destinations.

42. Farmer: Worldwide, just three grain crops-rice, wheat, and corn-account for most human caloric intake. To
maintain this level of caloric intake and also keep pace with global population growth, yields per acre from
each of these crops will have to increase at least 1.5 percent every year, given that the supply of cultivated
land is diminishing. Therefore, the government should increase funding for research into new ways to
improve yields.
Which of the following is an assumption on which the farmer’s argument depends?
(a) It is solely the government’s responsibility to ensure that the amount of rice, wheat, and corn produced
worldwide keeps pace with global population growth.
(b) Increasing government funding for research into new ways to improve the yields per acre of rice, wheat,
and corn crops would help to increase total worldwide annual production of food from these crops.
(c) Increasing the yields per acre of rice, wheat, and corn is more important than increasing the yields per
acre of other crops.
(d) Current levels of funding for research into ways of improving grain crop yields per acre have enabled
grain crop yields per acre to increase by more than 1.5 percent per year worldwide.

43. The air quality board recently informed Coffee Roast, a small coffee roasting firm, of a complaint regarding
the smoke from its roaster. Recently enacted air quality regulations require machines roasting more than 10
pounds of coffee to be equipped with expensive smoke-dissipating afterburners. The firm, however, roasts
only 8 pounds of coffee at a time. Nevertheless, the company has decided to purchase and install an
afterburner.
Which of the following, if true, most strongly supports the firm’s decision?
(a) Until setting on the new air quality regulations, the board had debated whether to require afterburners
for machines roasting more than 5 pounds of coffee at a time.
(b) Coffee roasted in a machine equipped with an afterburner has its flavour subtly altered.
(c) The cost to the firm of an afterburner is less than the cost of replacing its roaster with a smaller one.
(d) The firm has reason to fear that negative publicity regarding the complaints could result in lost sales.

44. People who do regular volunteer work tend to live longer, on average, than people who do not. It has been
found that “doing good,” a category that certainly includes volunteer work, releases endorphins, the brain’s
natural opiates, which induce in people a feeling of well-being. Clearly, there is a connection: Regular
releases of endorphins must in some way help to extend people’s lives.
Which of the following, if true, most seriously undermines the force of the evidence given as support for the
hypothesis that endorphins promote longevity?
(a) People who do regular volunteer work are only somewhat more likely than others to characterize the
work they do for a living as “doing good.”
(b) Although extremely high levels of endorphins could be harmful to health, such levels are never reached
as a result of the natural release of endorphins.
(c) There are many people who have done some volunteer work but who do not do such work regularly.
(d) People tend not to become involved in regular volunteer work unless they are healthy and energetic to
begin with.

45. A study compared a sample of Swedish people older than 75 who needed in-home assistance with a
similar sample of Israeli people. The people in the two samples received both informal assistance, provided
by family and friends, and formal assistance, professionally provided. Although Sweden and Israel have
equally well-funded and comprehensive systems for providing formal assistance, the study found that the
people in the Swedish sample received more formal assistance on average, than those in the Israeli
sample.
Which of the following, if true, does most to explain the difference that the study found?
(a) A companion study found that among children needing special in-home care, the amount of formal

14
assistance they received was roughly the same in Sweden as in Israel.
(b) More Swedish than Israeli people older than 75 live in rural areas where formal assistance services are
sparse or nonexistent.
(c) Although in both Sweden and Israel much of the funding for formal assistance ultimately comes from
the central government, the local structures through which assistance is delivered are different in the
two countries.
(d) In Israel, people older than 75 tend to live with their children, whereas in Sweden people of that age
tend to live alone.

46. Film Director: It is true that certain characters and plot twists in my newly released film The Big Heist are
similar to characters and plot twists in Thieves a movie that came out last year. Pointing to these
similarities, the film studio that produced Thieves is now accusing me of taking ideas from that film. The
accusation is clearly without merit. All production work on The Big Heist was actually completed months
before Thieves was released.
Which of the following, if true, provides the strongest support for the director’s position?
(a) Before Thieves began production, its script had been circulating for several years among various film
studios, including the studio that produced The Big Heist.
(b) The characters and plot twists that are most similar in the two films have close parallels in many earlier
films of the same genre.
(c) The film studio that produced Thieves seldom produces films in this genre.
(d) The director of Thieves worked with the director of The Big Heist on several earlier projects.

47. In Mernia commercial fossil hunters often sell important fossils they have found, not to universities or
museum, but to individual collector, who pay much better but generally do not allow researchers access to
their collections. To increase the number of fossils available for research, some legislators propose
requiring all fossils that are found in Mernia to be sold only to universities or museums.
Which of the following, if true, most strongly indicates that the legislators’ proposal will fail to achieve its
goal?
(a) Some fossil hunters in Mernia are not commercial fossil hunters, but rather are amateurs who keep the
fossils that they find.
(b) Most fossils found in Mernia are common types that have little scientific interest.
(c) Commercial fossil hunters in Mernia currently sell some of the fossils they find to universities and
museums.
(d) Most fossils are found by commercial fossil hunters, and they would give up looking for fossils if they
were no longer allowed to sell to individual collectors.

48. Economist: Tropicorp, which constantly seeks profitable investment opportunities, has been buying and
clearing sections of tropical forest for cattle ranching, although pastures newly created there become
useless for grazing after just a few years. The company has not gone into rubber tapping, even though
greater profits can be made from rubber tapping, which leaves the forest intact. Thus, some
environmentalists argue that Tropicorp’s actions do not serve even its own economic interest.
However, the initial investment required for a successful rubber-tapping operation is larger than that
needed for a cattle ranch; there is a shortage of workers employable in rubber-tapping operations; and
taxes are higher on profits from rubber tapping than on profits from cattle ranching. Consequently, the
environmentalists’ conclusion is probably wrong.
In the economist’s argument, the two boldface portions play which of the following roles?
(a) The first supports the conclusion of the economist’s argument; the second calls that conclusion into
question.
(b) The first states the conclusion of the economist’s argument; the second supports that conclusion.
(c) The first supports the conclusion of the environmentalists’ argument; the second states that conclusion.
(d) The first states the conclusion of the environmentalists’ argument; the second states the conclusion of
the economist’s argument.

15
49. Marketing executive for Magu Corporation: Whenever Magu opens a manufacturing facility in a new city,
the company should sponsor, or make donations to, a number of nonprofit organizations in that city. Doing
so would improve Magu’s image in the community, and thus the money spent on such charitable ventures
would lead to increased sales.
Which statement would, if true, point to the most serious weakness in the marketing executive’s advice?
(a) Magu sells its products internationally, so sales in any one city represent only a small portion of total
revenue.
(b) Spending on charitable ventures would require Magu to decrease direct advertisements, which are the
most effective means of reaching its target customers.
(c) If market conditions change, Magu may have to close any such facility or relocate it.
(d) Some nonprofit organizations are poorly organized, so money donated to them would be of little benefit
to the community.

50. Although the school would receive financial benefits if it had soft drink vending machines in the cafeteria,
we should not allow them. Allowing soft drink machines there would not be in our students’ interest. If our
students start drinking more soft drinks, they will be less healthy.
The argument depends on which of the following?
(a) If the soft drink vending machines were placed in the cafeteria, students would consume more soft
drinks as a result.
(b) The amount of soft drinks that most students at the school currently drink is not detrimental to their
health.
(c) Students are apt to be healthier if they do not drink soft drinks at all than if they just drink small amounts
occasionally.
(d) Students will not simply bring soft drinks from home if the soft drink vending machines are not placed in
the cafeteria.

51. Many athletes inhale pure oxygen after exercise in an attempt to increase muscular reabsorption of oxygen,
Measured continuously after exercise, however, the blood lactate levels of athletes who inhale pure oxygen
are practically identical, on average, to those of athletes who breathe normal air. The lower the blood
lactate level is, the higher the muscular reabsorption of oxygen is.
If the statements above are all true, they most strongly support which of the following conclusions?
(a) Athletes’ muscular reabsorption of oxygen is not increased when they inhale pure oxygen instead of
normal air.
(b) High blood lactate levels cannot be reduced.
(c) Blood lactate levels are a poor measure of oxygen reabsorption by muscles.
(d) The amount of oxygen reabsorbed by an athlete’s muscles always remains constant.

52. Historian: Fifteenth-century advances in mapmaking contributed to the rise of modern nation-states. In
medieval Europe (from the fifth to the fifteenth century), sovereignty centered in cities and towns and
radiated outward, with boundaries often ambiguously defined. The conceptual shift toward the modern state
began in the late fifteenth century, when mapmakers learned to reflect geography accurately by basing
maps on latitude-longitude grids. By the mid-seventeenth century, nearly all maps showed boundary lines.
Which of the following would if true, most strengthen the historian’s reasoning?
(a) Borders did not become condified in Europe until certain treaties were signed in the early nineteenth
century.
(b) During the medieval period, various authorities in Europe claimed power over collections of cities and
towns, not contiguous territories.
(c) Many members of the political elite collected maps as a hobby during the late sixteenth and early
seventeenth centuries.
(d) During the fifteenth century in Europe, mapmakers simplified the borders of sovereignty by drawing
clear lines of demarcation between political powers.

16
53. Sascha: The attempt to ban parliament’s right to pass directed-spending bills-bills that contain provisions
specifically funding the favourite projects of some powerful politicians-is antidemocratic. Our nation’s
constitution requires that money be drawn from our treasury only when so stipulated by laws passed by
parliament, the branch of government most directly representative of the citizens. This requirement is
based on the belief that exercising the power to spend public resources involves the ultimate exercise of
state authority and that therefore _________.
Which of the following most logically completes Sascha’s argument?
(a) designating funding specifically for the favourite projects of some powerful politicians should be
considered antidemocratic
(b) the right to exercise such a power should belong exclusively to the branch of government most directly
representative of the citizens
(c) exercising the power to spend public resources is in most cases-but not all-protected by the constitution
(d) modifications to any spending bills should be considered expenditures authorized by law

54. Boreal owls range over a much larger area than do other owls of similar size. Scientists have hypothesized
that it is scarcity of prey that lead the owls to range so widely. This hypothesis would be hard to
confirm directly, since it is not possible to produce a sufficiently accurate count of the populations of small
mammals inhabiting the forests where boreal owls live. Careful study of owl behaviour has, however,
shown that boreal owls do range over larger areas when they live in regions where food of the sort
eaten by small mammals is comparatively sparse. This indicates that the scientists’ hypothesis is not
sheer speculation.
In the argument given, the two boldfaced portions play which of the following roles?
(a) The first presents an explanatory hypothesis; the second states the main conclusion of the argument.
(b) The first presents an explanatory hypothesis; the second presents evidence tending to support this
hypothesis.
(c) The first presents an explanatory hypothesis; the second presents evidence to support an alternative
explanation.
(d) The first describes a position that the argument opposes; the second presents evidence to undermine
the support for the position being opposed.

55. Cognitive scientist: Using the pioneering work of comparative psychologist Gordon Gallup as a model,
several studies have investigated animals’ capacity for mirror self-recognition (MSR). Most animals
exposed to a mirror respond only with social behavior, such as aggression. However, in the case of the
great apes, repeated exposure to mirrors leads to self-directed behaviors, such as exploring the inside of
the mouth, suggesting that these animals recognize the reflection as an image of self. The implication of
these studies is that the great apes have a capacity for self-awareness unique among nonhuman species.
The cognitive scientist makes which of the following assumptions in the argument above?
(a) Gallup’s work has established that the great apes have a capacity for MSR unique among nonhuman
species.
(b) If an animal does not have the capacity for MSR, it does not have the capacity for self-awareness.
(c) If a researcher exposes an animal to a mirror and that animal exhibits social behavior, that animal is
incapable of being self-aware.
(d) When exposed to a mirror, all animals display either social behavior or self-directed behavior.

56. Last year a record number of new manufacturing jobs were created. Will this year bring another record?
Well, any new manufacturing job is created either within an existing company or by the start-up of a new
company. Within existing firms, new jobs have been created this year at well below last year’s
record pace. At the same time, there is considerable evidence that the number of new companies starting
up will be no higher this year than it was last year and there is no reason to think that the new
companies starting up this year will create more jobs per company than did last year’s start-ups. So
clearly, the number of new jobs created this year will fall short of last year’s record.
In the argument given, the two portions in boldface play which of the following roles?

17
(a) The first is a claim that the argument challenges; the second is an explicit assumption on which that
challenge is based.
(b) The first is a claim that the argument challenges; the second is a judgment advanced in support of the
main conclusion of the argument.
(c) The first provides evidence in support of the main conclusion of the argument; the second is an
objection that has been raised against that main conclusion.
(d) The first provides evidence in support of the main conclusion of the argument; the second is a judgment
advanced in support of that main conclusion.

57. A study of ticket sales at a summer theater festival found that people who bought tickets to individual plays
had a no-show rate of less than 1 percent, while those who paid in advance for all ten plays being
performed that summer had a no-show rate of nearly 30 percent. This may be at least in part because the
greater the awareness customers retain about the cost of an item, the more likely they are to use it.
Which of the following would, if true, best serve as an alternative explanation of the results of the study?
(a) The price per ticket was slightly cheaper for those who bought all ten tickets in advance.
(b) Many people who attended the theater festival believed strongly that they should support it financially.
(c) Those who attended all ten plays became eligible for a partial refund.
(d) Usually, people who bought tickets to individual plays did so immediately prior to each performance that
they attended.

58. Although there is no record of poet Edmund Spenser’s parentage, we do know that as a youth Spenser
attended the Merchant Tailors’ School in London for a period between 1560 and 1570. Records from this
time indicate that the Merchant Tailors’ Guild then had only three members named Spenser: Robert
Spenser, listed as a gentleman; Nicholas Spenser, elected the Guild’s Warden in 1568; and John Spenser,
listed as a “journeyman cloth-maker.” Of these, the last was likely the least affluent of the three-and most
likely Edmund’s father, since school accounting records list Edmund as a scholar who attended the school
at a reduced fee.
Which of the following is an assumption on which the argument depends?
(a) Anybody in sixteenth-century London who made clothing professionally would have had to be a
member of the Merchant Tailors’ Guild.
(b) The fact that Edmund Spenser attended the merchant Tailors’ School did not necessarily mean that he
planned to become a tailor.
(c) No member of the Guild could become Guild warden in sixteenth-century London unless he was a
gentleman.
(d) The merchant Tailors’ School did not reduce its fees for the children of the more affluent Guild
members.

59. Hea Sook: One should not readily believe urban legends. Most legends are propagated because the moral
lesson underlying them supports a political agenda. People will repeat a tale if it fits their purpose. They
may not deliberately spread untruths, but neither are they particularly motivated to investigate deeply to
determine if the tale they are telling is true.
Kayla: But people would not repeat stories that they did not believe were true. Therefore, one can safely
assume that if a story has been repeated by enough people then it is more likely to be true.
Kayla’s reply is most vulnerable to the criticism that it
(a) does not specify how many people need to repeat a story before someone is justified believing it
(b) overstates the significance of political agendas in the retelling of stories
(c) fails to address the claim that people will not verify the truth of a story that fits their purpose
(d) implicitly supports the claim that the people repeating legends are not deliberately spreading untruths

18
60. Rainwater contains hydrogen of a heavy form called deuterium. The deuterium content of wood reflects the
deuterium content of rainwater available to trees during their growth. Wood from trees that grew between
16,000 and 24,000 years ago in North America contains significantly more deuterium than wood from trees
growing today. But water trapped in several North American caves that formed during that same early
period contains significantly less deuterium than rainwater in North America contains today.
Which of the following, if true, most helps to reconcile the two findings?
(a) There is little deuterium in the North American caves other than the deuterium in the water trapped
there.
(b) Exposure to water after a tree has died does not change the deuterium content of the wood.
(c) Industrialization in North America over the past 100 years has altered the deuterium content of rain.
(d) Trees draw on shallow groundwater from rain that falls during their growth, whereas water trapped in
caves may have fallen as rainwater thousands of years before the caves formed.

CLAT 2020
New CLAT
PATTERN

OFFLINE TEST SERIES


• 20 OFFLINE MOCK TEST ` 5999/-
• 20 ONLINE MOCK TEST ENROLL
• CURRENT GK MAGAZINE (ONLINE)
• ONLINE TOPIC TEST
NOW
www. lawpreptutorial.com

• NEW PATTERN STUDY MATERIAL


ENQUIRY HELPLINE
076659 44999

19
Section III: Current Affairs

Directions for Questions 61 to 69: Which among the options (a), (b), (c) & (d) would best replace the
number given in brackets in the Paragraphs provided below.

The year 2019 saw [61] repeatedly provoking the U.S. and its allies. It shot down an American drone
over the Gulf in June, captured a British tanker in July and is believed to have either carried out or
orchestrated multiple attacks on oil tankers that pass through the [62] a narrow waterway that connects the
oil-rich Gulf with the Arabian Sea through the Gulf of Oman. In September, two Saudi oil facilities came
under attack, which temporarily cut the kingdom’s oil output by half. [61] was blamed for the attacks.
The attacks on Saudi facilities challenged the post-war partnership between the U.S. and Saudi Arabia that
guaranteed American protection to the kingdom. Still, the only counter-measure the U.S.
The U.S. is the world’s mightiest military power and arguably the centre of the post-Soviet world order. In
the 1990s, the U.S.’s dominance was at its peak with international and multilateral organisation getting
overshadowed by its pre-eminence. In 2001, after the September 11 terrorist attacks, it got international
support for its war in [63]. In 2003, the U.S. went ahead with the plan to bomb Iraq despite the UN
opposition, reminding the world of imperial invasions. But the global situation is different and more complex
today.

The North Atlantic Treaty Organization (NATO), the Cold War alliance that was formed as a
counterweight to the Soviet Union, continued to act as a vehicle of Western military dominance under the
leadership of the U.S. in the post-Soviet order. The alliance has come under pressure in recent years with
the rise of nationalist-populist leaders, including Mr. Trump, who have a favourable view of Russian
President [64] and are critical of NATO. These contradictions sharpened in 2019, suggesting that there are
growing cracks in the alliance. But the biggest crisis emerged when [65], the second largest military in
NATO, purchased S-400 missile defence system from Russia, NATO’s primary enemy and the main
geopolitical rival of the U.S., despite protests from the West.

61. (a) Bahrain (b) Iran (c) India (d) Russia

62. (a) Sunda Strait (b) Yucatan Strait (c) Strait of Hormuz (d) Malacca Strait

63. (a) Pakistan (b) Iran (c) Afghanistan (d) Syria

64. (a) Dmitri Mendeleev (b) Tuvsk Medvedev (c) Boris Yeltsi (d) Vladimir Putin

65. (a) Turkey (b) France (c) Canada (d) Germany,

The Karnataka byelection results have widely put to display the ineffectiveness of the [66]. Of the 17
defecting Congress-Janata Dal (Secular) MLAs, 11 were re-elected on a Bharatiya Janata Party (BJP)
ticket. Not only did this set of events lay down a well-structured framework to sidestep the law, it even set a
dangerous precedent for neutralising the consequences of the law altogether.

The phenomenon of defections is not new to Indian politics. It has is not new to Indian politics. It has
been plaguing the political landscape for over five decades. The most prominent case was that of
Haryana’s [67], originally an independent MLA who, in 1967, juggled between the congress and Janata
Party for two weeks, during which he switched his loyalty thrice.

The law originally protected the Speaker’s decision from judicial review. However, this safeguard was
struck down in [68]. While the SC upheld the Speaker’s discretionary power, it under-scored that the
Speaker functioned as a tribunal under the anti-defection law, thereby making her/his decisions subject to
judicial review. The same could be witnessed in [69], where the three-judge SC bench upheld the then
Karnataka Speaker’s decision of disqualification of the 17 rebel MLAs. However, it struck down his ban on
the MLAs from contesting elections till 2023, negating the only possible permanent solution to the problem.

20
66. (a) Anti Piracy Law (b) Anti-Defection Law (c) Anti Competition Law (d) Defection Law

67. (a) Gaya Lal (b) Gaya Ram (c) Gyanchand Khattar (d) Manohar Lal Khattar

68. (a) Indra Sawhney vs Union of India (b) Kihoto Hollohan v. Zachillhu and Others
(c) Maneka Gandhi vs Union of India (d) Kesavananda Bharati vs state of Kerala

69. (a) Shri Prabhunath Singh vs. Shri Ram Swaroop Prasad
(b) Shri Rajesh Verma vs. Shri Mohammad Shahid Akhlaque
(c) Shrimanth Balasaheb Patel & Ors vs Speaker Karnataka Legislative assembly
(d) ShriRajeev Ranjan Singh “Lalan” vs. Speaker Karnataka Legislative Assembly

70. Anti defection law relates to which schedule of constitution?


(a) 2nd (b) 9th (c) 5th (d) 10th

Directions for Questions 71 to 79: Which among the options (a), (b), (c) & (d) would best replace the
number given in brackets in the Paragraphs provided below.

On December 9, [71] announced a decision to ban [72] from global sporting competitions for a period
of four years. The 12-member executive committee voted unanimously to declare the Country’s Agency
“non-compliant” with global anti-doping rules. The move was a ratification of the findings by the
independent Compliance Review Committee (CRC) which had recommended that [72] be banned. The
watchdog’s decision is expected to affect the most at the coming up International Olympic summits [73]
and the [74] where the banned nation’s flag, name and anthem will not be allowed.

Though, IOC, under its president [75] has always opposed a blanket ban. He has maintained that he
favours “individual justice over collective punishment”. Apart from the Olympics, the ban can extend to the
2022 FIFA World Cup in [76].

71. (a) World Anti-Drug Agency (b) World Anti-Doping Agency


(c) World Agency against Joint (d) Worlds Anti Roll Agency

72. (a) India (b) Autralia (c) USA (d) Russia

73. (a) 2022 Beijing Olympic Games (b) 2020 Tokyo Olympic Games
(c) 2024 India Olympic Games (d) 1996 Beijing Olympic Games

74. (a) 2022 Beijing Winter Olympics (b) 2024 France Olympic Games
(c) 2024 India Olympic Games (d) 1996 Beijing Olympic Games

75. (a) Gianni Infantino (b) Henrietta Fore (c) Thomas Bach (d) Marco Lambertini

76. (a) Turkey (b) Qatar (c) UAE (d) Iran

The Climate Action Summit, and the Climate Youth Summit, succeeded in focusing the attention of
world leaders, from government, the private sector and civil society, on the urgency for action to address
the climate emergency, and on increasing climate action.
It was designed to be a different kid of Summit. Countries were also asked to present concrete and realistic
plans compatible with these goals. The Secretary-General asked leaders to come to the Summit to present
plans, not make speeches. The most ambitious plans were presented on 23 September, creating a unique
opportunity to showcase leadership towards transformative climate action that would make change in the
world.

21
Amid concerns that the collective actions of all nations together is not enough to save the world from ill
effects of global warming, the UN climate change conference kicked off in [77] on Monday where
representatives of nearly 200 countries will work on giving finishing touches to the rules governing the [78]
in next 11 days.
The twin objectives of [79] are to bring more and more nations on board to work for ‘net zero emissions’ by
2050 and finalise rules on how countries can reduce their emission using global carbon markets.

77. (a) Tibet (b) Beijing (c) Netherland (d) Madrid

78. (a) Paris accord (b) Kyoto Montreal Accord


(c) Vienna Agreement (d) Renaissance Agreement

79. (a) Conference (COP25) (b) Conference (COP28)


(c) Conference (COP24) (d) Conference (COP30)

80. The countries could not resolve outstanding issue at COP summit held last year. Where was the last
summit held?
(a) Poland (b) Hungary (c) Norway (d) Hague

81. ______, world’s top emitter of greenhouse gases, burns about half the coal used globally each year.
(a) India (b) China (c) Pakistan (d) Russia

82. Which of the following is not the mission of India’s National Action Plan on Climate Change?
(a) National solar mission (b) National water mission
(c) National Mission for a Green India (d) National Mission on Sustainable Development

Directions for Questions 83 to 89: Which among the options (a), (b), (c) & (d) would best replace the
number given in brackets in the Paragraphs provided below.

The Indians of Silicon Valley are the hidden geniuses of the tech revolution are Indian engineers–here’s
how one bucked stereotypes, got rich, and has become godfather to a generation of immigrant
entrepreneurs.

It’s safe to say that without Indian immigrants the Valley wouldn’t be what it is today. Indian engineers
have been coming to the U.S. in increasing numbers since the early 1970s; almost half the H-1B visas
given by the State Department go to Indian engineers (H-1Bs are granted to foreigners who have
specialized skills or are, oddly enough, fashion models). High-tech companies need people desperately–
U.S. engineering schools simply don’t produce enough graduates to fill the specialized jobs the high-tech
industry creates.

Recently, after the internet giant’s co-founders [83] and [84] stepped down from their executive roles,
making the India-born tech wizard one of the world’s most powerful corporate leaders. And [85] and [86] no
longer need two CEOs and a President. “The move confirms the ascendancy of [87], as one of tech’s most
powerful people”.

"Now he is the sole executive in charge of a company that has giant businesses in search, advertising,
maps, smartphone software and online video, as well as a variety of fledgling bets in far-off areas like drone
deliveries and internet-beaming balloons," it said.
Considered as a significant shakeup in the Silicon Valley company, the shuffle comes at a time when
company is facing mounting scrutiny over its size, data privacy practices and potential impact on society.
"I'm excited about our long term focus on tackling big challenges through technology. Thanks to founders,
we have a timeless mission, enduring values and a culture of collaboration & exploration - a strong
foundation we'll continue to build on," Neawly promoted head tweeted after the announcement of his
elevation.

22
83. (a) Jeff Bezos (b) Mark Zuckerbarg (c) Sundar Pichai (d) Larry Page

84. (a) Steve Jobs (b) Sergey Brin (c) Steve Wozniak (d) Ronald Wayne

85. (a) Alphabet (b) Apple (c) Facebook (d) Microsoft

86. (a) Amazon (b) Google (c) PayPal (d) YouTube

87. (a) Steve Jobs (b) Mr. Sundar Pichai (c) Bill Gates (d) Paul Allen

Success of Indian IT community abroad has motivated Engineering folks in India also. Recently [88],
founder CEO of India’s first private space start-up, [89] company says privatisation can create thousands of
jobs in the space sector. She has been named among BBC’s 100 Women of 2019. A self-confessed
“daughter of ISRO” and protégée of Arthur C. Clarke, the “anti-status-quo-ist”

88. (a) Radha Pillai (b) Susmita Mohanty (c) Ankiti Bose (d) Ananya Pandey

89. (a) SpaceX (b) Saturn Rings (c) Akash Ganga (d) Earth2rbit

Directions for Questions 90: Which among the options (a), (b), (c) & (d) would best replace the number
given in brackets in the Paragraphs provided below.

The high court had dismissed the 74-year old senior Congress leader’s bail plea in the ED case, saying
prima facie allegations against him were serious and he played an “active and key role” in the offence. It
had said that the entire community is aggrieved if offenders, who ruin the economy of the State, are not
brought to books.

Former Minister had sought bail in the high court, saying as the evidence is documentary and in the
custody of probe agencies, he cannot tamper with it. The ED had opposed his plea on the ground that he
has tried to influence and threaten witnesses.

He was first arrested by the Central Bureau of Investigation (CBI) on August 21 in the INX Media
corruption case and was granted bail by the Supreme Court on October 22. He was arrested by the ED in
the money laundering case on October 16 and is in judicial custody till November 27 under the order of a
trial court.

The Delhi High Court directed the AIIMS director to constitute a medical board on Thursday itself to
give opinion on the health of former finance minister [90], who is suffering from Crohn’s disease and was
lodged in Tihar jail in the INX Media money laundering case.

ED is responsible for enforcement of the Foreign Exchange Management Act, 1999 (FEMA) and certain
provisions under the Prevention of Money Laundering Act (PMLA).
The Directorate is under the administrative control of Department of Revenue (under the Ministry of
Finance) for operational purposes. The Directorate enforced regulations under the Foreign Exchange
Regulation Act, 1973 but later on FERA is being replaced by the FEMA.

90. (a) P Chidambaram (b) Pranab Mukherjee (c) Ravikrishna Rai (d) Karunanidhi Kanimozhi

91. Who was the first finance minister of India?


(a) Radha Krishanan (b) Jawahar lal Nehru (c) Ramasamy Chetty (d) P Annillai

92. Who among the following is first woman finance minster of India holding the office independently?
(a) Nirmala Sitharaman (b) Indira Gandhi (c) Anni Krishnan (d) Kanimozhi

23
93. ED as mentioned in passage stands for
(a) Economy Director (b) Enforcement Directorate
(c) Environment Directorate (d) Endowment Directorate

94. The authorization for the withdrawal of funds from the consolidated Fund of India must come from:
(a) The President of India (b) The Parliament of India
(c) The Prime Minister of India (d) The Union Finance Minister

95. Which of the following documents are presented to the legislature along with the budget?
I. An explanatory memorandum on the budget II. A summary of demands for grants
III. An Appropriation Bill IV. A Finance Bill
V. The economic survey
Code:
(a) I, III and V (b) I, II and III (c) II, III and V (d) I, II, III and IV

Directions for Questions 96 & 97: Which among the options (a), (b), (c) & (d) would best replace the
number given in brackets in the Paragraphs provided below.

The removal of Articles 370 and 35A of the Constitution within 10 weeks of the new government has
come as step towards realising the dreams of Sardar Vallabhbhai Patel, said Prime Minister Narendra Modi
today.
PM Modi said this while addressing the nation from the rampart of Red Fort in Delhi on the occasion of
Independence Day.
He said, "The new government has not even completed 10 weeks in office but even in this small period we
have taken and strengthened initiatives in all directions."
"The old arrangement in Jammu, Kashmir and Ladakh encouraged corruption, nepotism but there was
injustice when it came to rights of women, children, Dalits, tribal communities. The dreams of sanitation
workers were incomplete. How can we accept such a situation,"PM Modi said.
"Those who supported Article 370, India is asking them: If this was so important and life changing, why was
this Article not made permanent. After all, those people had large mandates and could have removed the
temporary status of Article 370,"Prime Minister Modi said.
The union territories of Jammu and Kashmir, and Ladakh, have been carved out of the state of Jammu and
Kashmir, in accordance with the government's August 5 decision to revoke the special status of the state
under Article 370 and bifurcate it into two Union Territories.
Following the abrogation of article 370, the menace of terrorism and its total elimination is one aspect which
is on the highest priority of the government. Amit Shah’s recent statement in response to a query
concerning normalcy in Jammu and Kashmir after abrogation of article 370 endorses government’s concern
and commitment towards eradicating terror but simultaneously has raised eyebrows of many , who still are
skeptical of this historic move. He further asserted that not even a single bullet has been fired since August
5, 2019. But on intricately analysing the scenario Post article 370, one finds that the statement is definitely
true to some extent but at the same time there have been a large number of terror related incidents in
Kashmir valley.
Stepping in-to the record books as the first lieutenant governors of two new union territories created after
the abrogation of 370 and 35 A, [96] was on Thursday sworn in as lieutenant governor of Jammu and
Kashmir and [97] of Ladakh.

96. (a) Jagdeep Dhankar (b) Anandiben Patel (c) Satya Pal Malik (d) G C Murmu

97. (a) R K Mathur (b) Arif Mohammad (c) Shri Kalraj Mishra (d) Shri Ganga Prasad

98. The oath of office to the governor is administered by:


(a) The President of India (b) The Chief Justice of the concerned state high court
(c) The Chief Justice of the Supreme Court (d) None of the above

24
99. In case of death of a sitting governor, who among the following will discharge the functions of the governor
of that state?
(a) The President of India
(b) The Chief Justice of the Supreme Court
(c) The Chief Justice of the concerned state high court
(d) None of the above

100. Which of the following powers are not possessed by a Governor?


(a) Diplomatic Powers (b) Executive powers
(c) Judicial powers (d) Legislative powers

ADMISSION OPEN
CLAT
2020 CRASH COURSE
w w w. l a w p r e p t u t o r i a l . c o m

Starting from: 18, 23, 26, 31 March & 1 April


ENROLL NOW

25
Section IV: Legal Reasoning

Supreme Court (SC) said it will hear all pleas related to the Jamia Milia Islamia clashes while issuing a
stern warning to students to stop riots. Citing destruction of property amid anti-Citizenship Amendment Act
protests in the national capital, Chief justice of India SA Bobde said, If violence and destruction of public
property continues, we will not hear it. The development comes as several pleas have been filed after a
violent clash between Delhi Police and Jamia students. Pleas on similar clashes in Aligarh Muslim
University will also be heard by the Supreme Court. Meanwhile, protests were carried out in several places
in West Bengal and Assam. At least 5 trains and three railway stations were torched in West Bengal.
Similar scenes were witnessed in Assam as well.
A large number of citizens are on the streets protesting the newly amended Citizenship Act. Their right
to protect emerges from Article 19 (1)(a) of the Constitution. It gives all citizens the right “to assemble
peacefully and without arms”. Citizens hitting the streets of cities across states against the Citizenship
Amendment Act are exercising their fundamental right guaranteed under the Constitution. But the visuals
beamed on the television show that many of them are using arms – stones, bricks, lathis and some
inflammable materials too and have damaged public property, even resorting to setting public and private
vehicles on fire. These violent protests have taken place at places where restrictions were imposed under
Section 144 of the Criminal Procedure Code (CrPC) which restricts people from coming in gatherings.
Resorting to violence during protest is violation of a key fundamental duty of citizens. Enumerated in Article
51A, the Constitution makes it a fundamental duty of every citizen “to safeguard public property and to
abjure violence”.
The Prevention of Damage to Public Property Act, 1984 punishes anyone “who commits mischief by
doing any act in respect of any public property” with a jail term of up to five years and a fine or both.
Article 246 of the Constitution places ‘public order’ and ‘police’ under the jurisdiction of the state. This
gives each state government full legislative and administrative powers over the police. The Ministry of
Home Affairs (MHA) can deploy Central Armed Police Forces (CAPFs) to the state to assist the state civil
police and armed forces.
Public property under this Act includes “any building, installation or other property used in connection
with the production, distribution or supply of water, light, power or energy; any oil installation; any sewage
works; any mine or factory; any means of public transportation or of telecommunications, or any building,
installation or other property used in connection therewith”. However, the Supreme Court has on several
earlier occasions found the law inadequate, and has attempted to fill the gaps through guidelines.
In 2007, the court took suo motu cognizance of “various instances where there was large scale
destruction of public and private properties in the name of agitations, bandhs, hartals and the like”. A three-
bench of the Supreme Court ruled, “Persons who have initiated, promoted, instigated or any way caused to
occur any act of violence against cultural programmes or which results in loss of life or damage to public or
private property either directly or indirectly, shall be made liable to compensate the victims of such
violence.”
101. What is the view of Supreme Court regarding damage caused to private and public property due to violent
protests?
(a) The Supreme Court held that those giving call for protests should be made liable for damage to public
property.
(b) Court held that freedom of speech is allowing citizens, for one, the right “to assemble peaceably and
without arms.” This includes the right to form associations, hold meetings, and come out in processions.
(c) The court has held innocent people calling for protests cannot be made liable to pay for the damage
(d) Court held Constitutional right to assembly is, however, subject to certain regulations contained in a
number of laws, such as the Indian Penal Code, the Criminal have Procedure Code, and the Police Act
of 1861.

26
102. If the protesters often argued that they did not give a call for violence and those resorting to violence
were “outsiders”. As per the reasoning in the above passage what would be the courts view in such
incident?
(a) Court would hold that flexibility of discourse is permitting citizens right “to gather serenely and without
arms.” This incorporates the right to create affiliations, hold gatherings, and come out in processions.
(b) Court would hold that those giving call for protest ought to be made at risk for harm to public property.
(c) Court would hold that who commits mischief by doing any act in respect of any public property must be
jailed.
(d) Resorting to violence during protest is violation of a key fundamental duty of citizens.

103. As per the above passage does Supreme Court think that Indian law sufficiently deals with cases and
situation where damage is caused to a public property due to violent protest?
(a) Laws empower the government to impose certain “reasonable restrictions” on the right to assemble, if
such assembly is likely to lead to a disturbance in public peace and order or if it poses a threat to
national sovereignty.
(b) Constitution seeks a balance between the freedom of speech guaranteed in Article 19 (1) (b) and social
order as defined in Article 19 (3). The police also have the duty to control and regulate crowds while
providing citizens the space and peace to exercise their right to assembly.
(c) Each state’s police force has two components: the civil police and the armed police. While the civil
police control crime, the armed police are specialised police units that deal with extraordinary law and
order situations. Although matters of the police are a state subject, the Constitution empowers the
central government to intervene in certain police matters in order to protect the state in times of
emergency.
(d) Court has on a few prior events found the law lacking, and has endeavoured to fill the crevices through
rules.

104. There are times when the protest takes a violent turn, either among the protesters or between them and the
police. The risk of a protest turning violent has increased in recent times. What can MHA do to deal with
sudden grave situation of peace and order in a State?
(a) MHA would be useless in such situation because there job is to secure Indian border and deal with
situations of terrorism only.
(b) MHA would only be assisting a state if the State Government and Central Governments are of same
party.
(c) MHA can deploy Central Armed Police Forces (CAPFs) to the state to assist the state civil police and
armed forces.
(d) MHA can do nothing as peace and order is matter falling under State list. Thus, Central Government
and its Ministry are not supposed to deal with these situations.

105. As per the given passage which all provisions are available to deal with the right to protests and damage
caused by the protests?
(a) Article 19 (1)(a) of the Constitution. It gives all citizens the right “to assemble peacefully and without
arms”.
(b) Article 51A, the Constitution makes it a fundamental duty of every citizen “to safeguard public property
and to abjure violence”
(c) The Prevention of Damage to Public Property Act, 1984 punishes anyone “who commits mischief by
doing any act in respect of any public property” with a jail term of up to five years and a fine or both.
(d) All of the above.

106. In February 2016, the state of Haryana experienced a lockdown as members of the Jat community sought
political and economic reservations from the government. In parts of the state, protesters clashed with
security forces, killing over 30 people.
As per the given passage will Supreme Court be non critical of this protest?

27
(a) No, in a country like India characterised by a number of religious, ethnic, language and caste divisions,
the police are regularly confronted with large-scale protests, riots and incidents of mass civil disorder.
(b) No, Court is completely against this kind of protest which is violent in nature.
(c) Yes, in such events of unrest, protesters may react in a violent way against police and security forces,
property, or other civilians.
(d) Yes, it is essential to discuss and deliberate on the ways and means to deal with violent civil protests.

107. The death of militant leader Burhan Wani sparked mass civilian protests in Kashmir. Security forces were
criticised for their use of excessive force against a crowd that engaged in stone-pelting and arson. As per
the given passage will Supreme Court be non critical of this protest?
(a) No, the government should re-examine the methods and tactics used by the police when dealing with
civil unrest
(b) No, Court is completely against this kind of protest which is violent in nature. Like the case in passage
Supreme Court (SC) said it will hear all pleas related to the Jamia Milia Islamia clashes while issuing a
stern warning to students to stop riots.
(c) No, the conversation must focus on understanding the lessons learnt from civilian protests that begin
peacefully but eventually regress to violence and result in high casualties
(d) Yes, in India, police have used lathis or batons, water cannons and teargas to disperse an unlawful
assembly.

108. Monday saw DMK party president M.K. Stalin and DMK Lok Sabha member M. Kanimozhi tweeting
photographs of the outside of their residences that had kolams with messages against the CAA, such as
‘Vendaam CAA’. A kolam also appeared in front of late Chief Minister M. Karunanidhi’s Gopalapuram
residence. As per the given passage will Supreme Court be non critical of this protest?
(a) No, the government should re-examine the methods and tactics used by the police when dealing with
civil unrest and arrest these kolam artists.
(b) No, Court is completely against this kind of protest which is peaceful in nature. Like the case in
passage Supreme Court (SC) said it will hear all pleas related to the Jamia Milia Islamia clashes while
issuing a stern warning to students to stop drawing kolams.
(c) Yes, Court is only against the protests indulging or causing violence such peaceful protests are not
causing any damage to anyone.
(d) Yes, Citing destruction of property amid anti-Citizenship Amendment Act protests in the national capital,
Chief justice of India SA Bobde said, If violence and destruction of public property continues, we will
hear the plea filed against CAA.

109. As per the given passage which of the following ways of protest would be justified?
(a) To avoid paying the British tax on salt, Gandhi decided to get his own salt. To do this, he walked 240
miles over the course of 24 days, joined by a growing number of followers.
(b) To get Independence from the former Soviet Union for Estonia, Latvia and Lithuania, Protesters
gathered in the streets where they sang songs of national pride, which had been outlawed by the Soviet
occupiers.
(c) To stop deforestation of the Pureora forest in New Zealand. Protestors Built tree houses, refused to
leave them
(d) All of the above

One of the consequences of living in a digital era is that a court ruling in one part of the world may
trigger a reaction elsewhere. Following the ruling of the Court of Justice of the European Union against the
search engine Google in the “Right to be Forgotten” case, there has been a spike in takedown requests. I
have been receiving mails from readers, PR agencies, corporates and litigants seeking the removal of a
story from The Hindu archives.

28
First, let’s look at the background to the European Union’s Court ruling. A Spanish citizen, Costeja
González, was in financial trouble way back in 1998, and his home loan foreclosure notices appeared in a
Spanish newspaper La Vanguardia. But because the original purpose of the notice to attract a potential
buyer had lapsed, he wanted the newspaper to remove them. He lodged a complaint with the Spanish Data
Protection Agency, the AEPD against the newspaper and against Google. The AEPD rejected the
complaint against La Vanguardia, taking the view that the information in question had been lawfully
published by it. On the other hand, the complaint was upheld as regards Google Spain and Google Inc, and
the two companies were requested “to withdraw the data from their index and to render access to the data
impossible in the future.” The case was escalated to the Spanish National Court, and later referred to the
Court of Justice of the European Union. Although, this right is in sync with right to privacy, the primary
arguments made against the ‘right to be forgotten’ were from its conflict with the right to freedom of speech.
The court ruled that even if the physical server of a company processing data is located outside
Europe, EU data protection law applies and so does the right to be forgotten. Thus, individuals do have the
right under certain conditions to ask search engines to remove links with personal information about them.
However, it maintained that the right to be forgotten is not absolute and it always needs to be balanced with
other fundamental rights, such as the freedom of expression and of the media.
The governing policy is to neither withdraw nor alter the content when the reporting itself was not in
error or vitiated by any other factors. As the issue here is one of Internet search engines bringing up the
material from a link, it essentially needs to be addressed to the search engines, and not the publisher of the
material.

110. Which of the following views can be correctly attributed to the author of the above passage?
(a) The newspaper, must consider follow-up reporting if any subsequent developments warrant one for a
report published earlier.
(b) Reports that have been published are not necessarily factual accounts of events and thus they should
be removed.
(c) The archives section of the newspaper’s website is a not a repository of reportage and articles
published at different points of time.
(d) The right to be forgotten empowers individuals to ask organisations to delete their personal data from
search engines but the newspaper doing their job of reporting are not bound by it until and unless there
is fundamental fault in the reporting

111. As per the rationale of rulings of the EU court the author could be required to remove which of the following
news:
(a) News about companies who lost out following the de-allocation of coal blocks by the Supreme Court.
(b) News about a builder wanting to remove the name of the building in which a fatal accident happened.
(c) Another writer wanted to take down a report of a court proceeding, as the case was later resolved
through an out-of-court settlement.
(d) None of the above

112. Suppose the Right to be forgotten refers to the ability of an individual to limit, delink, delete, or correct the
disclosure of the personal information on internet or anywhere that is misleading, embarrassing,
unnecessary or irrelevant. In that case would a rape victim name be removed from the case files.
(a) Yes, in sensitive cases involving women in general and highly sensitive cases involving rape or
affecting the modesty and reputation of the person concerned the rape victim could claim right to be
forgotten as it would result into future unnecessary humiliation to her
(b) Yes, This is a matter of her right to privacy.
(c) No, whether the data online has to be retained (right to information) or erased (right to be forgotten)
from the web, the decision has to be taken by some Authority.
(d) No, other citizens right to information will be violated.

29
113. Suppose the Right to be forgotten refers to the ability of an individual to limit, delink, delete, or correct the
disclosure of the personal information on the internet or anywhere that is misleading, embarrassing, or
irrelevant. Can a criminal claim that she has the right to insist that his conviction should not be referred to
by the media?
(a) Yes, this is embarrassing, or irrelevant information which can create problems for the convict in her
future. Thus, she has right to be forgotten.
(b) Yes, in sensitive cases involving women in general convict could claim right to be forgotten as it would
result into future unnecessary humiliation to her
(c) No, This is not misleading, embarrassing, unnecessary or irrelevant information rather the conviction of
a criminal is a matter of fact here which would be required to be published in public interest.
(d) No,

114. Suppose the Right to be forgotten refers to the ability of an individual to limit, delink, delete, or correct the
disclosure of the personal information on the internet anywhere that is misleading, embarrassing, or
irrelevant. A politician promised certain projects to its constituency and thus is not able to fulfil them after
years of struggle want to get all those promises erased as he was not able to fulfil them due to unavoidable
circumstances. Can he claim right to be forgotten?
(a) Yes, the politician wanting to change his promises as per practical purposes should be allowed to do
so.
(b) Yes, the non fulfilment of promises and their reminder will cause continuous humiliation to him.
(c) No, the freedom to criticise the public personalities for their public policies based on their past
statements and activities will be in jeopardy
(d) No, This is a relevant information concerning public interest. Thus, right to be forgotten would not apply.

115. Right to be forgotten is going to be in direct restriction of the following right:


(A) Right to Knowledge (B) Right to Remember
(C) Right to Privacy (D) Right to Speech

The demand for speedy retributive justice in the recent heinous crime done against a veterinarian has
brought into light the question of extra-constitutional killings. The public sentiments, political demand of
public lynching of rapists inter-alia have raised the debate whether a democratic country should follow the
constitutional norms and adhere to the due process of law or shall it adopt the measures of retributive
justice to bring instant and speedy justice to the victim.
Retributive justice is a system of criminal justice based on the punishment of offenders rather than on
rehabilitation where as in REFORMATIVE THEORY the object of punishment should be the reform of the
criminal, through the method of individualization. It is based on the humanistic principle that even if an
offender commits a crime, he does not cease to be a human being.
From protests on the ground, to the commentary on social media, to MPs in Parliament, the demand for
the instant killing of the accused from all corners created the public opinion for the abandonment of the rule
of law that appears to have led to the incident.
Justice in any civilised society is not just about retribution, but also about deterrence, and in less serious
crimes, rehabilitation of the offenders.
There is a procedure prescribed by the law for criminal investigation which is embedded in
constitutional principles. Article 21 of the Constitution (which is fundamental and non-derogable) states that
no person shall be deprived of his life or personal liberty except according to the procedure established by
law.
Also in the Salwa Judum case in 2011 a core constitutional precept was set out that in modern
constitutionalism no wielder of power can be allowed to claim the right to perpetrate state’s violence against
anyone. This is also the touchstone of the constitutionally prescribed rule of law (Article 14).

30
Hence, it is the responsibility of the police, being the officers of government, to follow the Constitutional
principles and uphold the Right to Life of every individual whether an innocent one or a criminal.
According to Dr. B.R. Ambedkar, the pathways of justice are not linear nor without obstacles. But we
have, as a people, chosen the route of democracy and the Constitution, so we really have no option but to
school ourselves in constitutional morality that with time must replace public morality
This passage is based on “Demonising the legal system won’t help” which was published in The
Hindu on 09/12/2019.

116. Which of the following views can be correctly attributed to the author of the above passage?
(a) The political sanction of “encounter killings” to deliver swift retribution would only be a disincentive for
the police to follow due process and may even deter them from pursuing the course of justice. Far from
ensuring justice to the victims, bending the law in such cases would only increase people’s faith in the
criminal justice system.
(b) The ends of justice are not served by wanton killing and retributive measures. Justice lies in supporting
them in their moment of grief and pain and insisting on due process that brings suspects and accused
to trial through a robust, stringent and competent criminal investigation.
(c) For several affected families, death is the only answer to rape. The rule of law is not giving women the
due justice, and that at the very least, laws need to be amended to create a stronger deterrent and
provide quicker closure to victims of crimes of sexual violence.
(d) Retributive justice is the way to go in a democratic country as legal institutions are ill-equipped to deal
with such crimes and to bring the perpetrators to justice.

117. What does the author mean by Retributive Justice?


(a) It is a theory of punishment that when an offender breaks the law, justice requires that he or she must
suffer equally or more in return.
(b) It requires that the response to a crime must be proportionally less to the committed offence.
(c) There is a need to keep in sight of the rule of law and constitutional tenets.
(d) None of the above

118. As per author what is reformative theory?


(a) This theory believes a criminal will always remain a criminal. Thus, society should reform itself for
dealing with this criminal.
(b) The object of punishment should not be to bring about the moral reform of the offender. As existing
laws on sexual crimes and punishment need a better application, a recourse to brutal retribution as
suggested wisely by many is solution.
(c) Offender must not be educated and taught some art or industry during the period of his imprisonment
so that he may be able to start his life again after his release from jail.
(d) An effort should be made to reform offender during the period of his incarceration.

119. As per author the extrajudicial killing of those 4 convicts was a form of
(a) Retributive Justice (b) Reformative Justice
(c) Democratic Justice (d) No Justice

120. As per author retributive justice could be done in following circumstances:


(a) when police is dealing with gruesome offender of cannibalism The political sanction of “encounter
killings” to deliver swift retribution would only be a incentive for the police to follow due process and
may even motivate them from pursuing the course of justice.
(b) when a minor child is brutally raped. There is no doubt that the anguish or outrage of people in the
aftermath of such horrific crimes would be satisfied by encounter killing.
(c) when there is possibility of extreme delay in delivery of justice, bending the law in such cases would
only increase people’s faith in the criminal justice system.
(d) none of the above

31
Seed is the primary input of a productive agriculture set-up. No one can deny the fact that the seed
industry has been at the centre of the significant advances made in agriculture in the last four decades and
will continue to do in the years to come. The seed industry welcomes the prompt progress made by the
Government in finalising the draft of the Seed Bill 2019 which is likely to be introduced in Parliament soon.
It is an important legislation to ensure supply of modern, high quality, cutting edge seed technologies to the
farmers which will help them in enhancing their productivity and profitability.
The seed industry has made certain observations and recommendations to the government based on
the draft of the Bill released by the Ministry of Agriculture. Some of them are listed below.
There must be a system of accreditation of national level research-based companies with integrated
facilities for research, product testing, data analytics, seed production, seed quality control, seed
processing, farmer extension and marketing.
The industry welcomes the proposed mandatory registration of seed varieties as it will bring greater
accountability to seed companies. However, this should be made time bound. Also, pure export-oriented
varieties must be exempt from registration.
The new Bill proposes registration /licences for seed companies, seed processing plants, seed
producers, seed dealers and nurseries which is a good measure. However, this should apply to all
nurseries and not just fruit nurseries as proposed.
An important clause in the draft is the powers of the government to fix prices of selected varieties in
case of ‘emergent’ situations. The situations are defined as those of seed shortage, abnormal increase in
price, monopolistic pricing, profiteering, etc., which are open to subjective interpretation. The industry is
opposed to any kind of price control, as it will stifle innovation and result in a scale back of research
investments as happened in the case of cotton. A competitive seed industry with more than 400 companies
have kept prices low, and seeds account for less than 5 per cent of the total cultivation cost of farmers.
There is really no reason to give these powers to the government.
The industry has made a recommendation to differentiate between minor offences, unintended offences
and major offences made intentionally. It is not fair to use criminal proceedings for all offences. There
should be a provision for compounding of minor offences.
(https://www.thehindubusinessline.com/opinion/the-seed-bill-needs-to-be-tweaked-to-serve-the-interests-of-
all-stakeholders-better/article30095614.ece#)
121. Which of the following views can be correctly attributed to the author of the above passage?
(a) The proposals relating Seed Bill 2019 is as it is effective and industry friendly
(b) The proposals relating Seed Bill 2019 needs some alterations to make the law more effective and
industry friendly
(c) The proposals relating Seed Bill 2019 needs complete transformation as the law as proposed is of no
use to seed industry
(d) The proposals relating Seed Bill 2019 needs no alterations.

122. Based on the seed industry’s arguments in the passage above, which of the following would be most
correct:
(a) The state should not control prices of certain varieties, since this would hamper the development of
seed companies, as it will stifle innovation and result in a scale back of research investments
(b) The state should control the prices of certain varieties, since it would also be manufacturing them itself.
(c) The state should not control the prices of certain varieties, since the market would determine the best
price for such varieties.
(d) The state should control the prices of certain varieties, since citizens are likely to use them more often.

123. An essential seed variety, ‘Basmati Wheat’, is used to commonly in many households, and there is only
one company engaged in its sale. If this is true, then, based on the author’s reasoning in the passage
above:
(a) The state should not control the price of Basmati Wheat, since it should allow the market to function.
(b) The state should control the price of Basmati Wheat, since there is no competition in the marketplace,
and it cannot function to control the price of this variety.
(c) The state should ban the sales of Basmati Wheat, and ask people to consider using some other variety
that is made by more than one manufacturer.
(d) The state should take up the manufacture of Basmati Wheat itself.

124. Shakal has an apple varieties nursery (fruit nursery). Based on the seed industry’s arguments in the
passage above, which of the following would be most correct:
(a) Shakal is against the whole proposed bill and wants nothing to do with Seed Bill
(b) Shakal should be free not to register as that is a good measure for him being a fruit nursery.
(c) Seed Industry has appreciated the bill and does not want to include fruit nurseries in the registration
and license process.
(d) Shakal being a fruit nursery would be made to apply for registration and license.

125. RK Industries is in business of developing seed varieties for solely export purposes. Based on the seed
industry’s arguments in the passage above
(a) RK Industry must pay more tax to government as being export oriented it must be doing good business.
(b) As per seed Industry’s recommendation RK Industries must not be required to register as pure export-
oriented varieties must be exempt from registration.
(c) RK Industries must register as the industry welcomes the proposed mandatory registration of seed
varieties as it will bring greater accountability to seed companies.
(d) Overall, the seed law should have the twin objective of regulating the supply of seeds for the benefit of
the farmer and, at the same time, enable the development of the seed industry

126. Based on the seed industry’s arguments in the passage above, which of the following would not be correct:
(a) The industry welcomes the proposed mandatory registration of seed varieties as it will bring greater
accountability to seed companies.
(b) The industry welcomes proposed Government control over price of seed varieties in most
circumstances.
(c) Seed industry believes the seed law should have the twin objective of regulating the supply of seeds for
the benefit of the farmer and, at the same time, enable the development of the seed industry
(d) There must be a system of accreditation of national level research-based companies with integrated
facilities for research, product testing, data analytics, seed production, seed quality control, seed
processing, farmer extension and marketing.

Questions about a person’s place and date of birth, his parents’ names and their place of birth are
meant to ascertain citizenship, the Supreme Court has held. The court, in a 2005 judgment has clearly
held that these personal questions are directly associated with “establishing citizenship”. “In order to
establish one’s citizenship, normally he may be required to give evidence of (i) his date of birth (ii) place of
birth (iii) name of his parents (iv) their place of birth and citizenship,”. Moreover, the court explained that
these facts figured specially in the context of establishing citizenship because they “would necessarily be
within the personal knowledge of the person concerned and not of the authorities of the State”. In case of
doubts about a person’s citizenship, the burden of proving that these facts were true was on the person
concerned.
The government has so far denied any link between the National Population Register (NPR), which is
to establish usual residency, and the National Register of Citizens (NRC), meant to establish citizenship. In
fact, the NPR takes into account foreign citizens too. However, if the NPR form of 2020 carries these
personal questions highlighted in the judgment, apprehensions raised in the public mind that the NPR is a
stepping stone for a nationwide NRC become justified. The 2005 judgment came just over a year after the
Citizenship (Registration of Citizens and Issue of National Identity Cards) Rules of 2003 was notified in
December 2003. The Rules clearly linked the NPR and the NRC. Rule 4(3) of the Citizenship Rules states
that personal details collected for the ‘Population Register’ would be used in the preparation of the National

33
Register of Indian Citizens. The Citizenship Rules define ‘Population Register’ as a “register containing
details of persons usually residing in a village or rural area or town or ward or demarcated within a ward in
a town or urban area”.
127. Based on the author’s arguments in the passage above, which of the following would be most correct:
(a) Questions about a person’s place and date of birth, his parents’ names and their place of birth are
meant to ascertain citizenship, the Supreme Court has held. The Rules also clearly linked the NPR and
the NRC.
(b) NPR is the last step towards conducting a nationwide NRC is mentioned in the Citizenship
(Registration of Citizens and Issue of National Identity Cards) Rules, 2003 under the Citizenship Act
1955.
(c) "some" data collected for the National Population Register (NPR) " may not be used" for the
implementation of a nationwide National Register of Citizens (NRC).
(d) Author is not about the chronology of events regarding NPR and NRC.

128. Does the author agree with government’s point that NPR and NRC are not linked.
(a) Yes, The author believes that government is right when the say that there is no connection between
NPR and NRC.
(b) Yes, author agrees with the government’s point as arguments given by author are contradictory.
(c) No, the author through judgement and rule is drawing a connection between NPR and NRC. Thus the
author does not agree with government’s opinion.
(d) Yes because author links NPR and NRC on the basis of forms which are yet to come to create a
relation.

129. As per the Courts Judgement for the purpose of citizenship which of the following are possible evidences:
(a) His date of birth
(b) Place of birth
(c) Name of his parents and their place of birth and citizenship
(d) All of the above

130. Due to so much concern about citizenship, NPR and NRC when Rahul went to meet his friend Amit’s house
he took all his documents with him. Siddhu stole all the documents because he wanted to blackmail Rahul.
Keeping in mind as per courts judgement, which of the following would be most correct:
(a) Siddhu has the burden of proof to prove that she not anti national.
(b) Rahul has the burden of proof to prove his citizenship.
(c) Rahul need not prove his citizenship because he is born in India, lived in India and will die in India.
(d) Amit should prove Rahul’s citizenship because he supports government’s rule for citizenship.

131. Based on the author’s arguments in the passage above, which of the following would be most correct:
(a) National Population Register (NPR), which is to establish citizenship, and the National Register of
Citizens (NRC), meant to establish usual residency.
(b) National Population Register (NPR), which is to establish usual residency, and the National Register of
Citizens (NRC), meant to establish citizenship.
(c) NPR and NRC both are meant to divide India.
(d) All the previous government as well as the present one are twisted for keeping India limited to Indian
citizens only.

132. Based on the author’s arguments in the passage above, which of the following would be most correct to
justify the public apprehensions:
(a) If the NPR form of 2020 carries the personal questions highlighted in the judgment
(b) If the NPR form of 2020 does not carries the personal questions highlighted in the judgment
(c) If the government announces there is no link between the NPR NRC
(d) In reality public is only concerned about basic amenities rather than NPR/NRC

34
The age of superannuation in the CAPFs has been a bone of contention for many years. Among the
CAPFs a differential treatment was given to those belonging to the AR and the CISF, where all members in
all ranks retire at the age of 60. But, in the BSF, CRPF and ITBP two classes have been created where
officers above the rank of Commandant retire at 60 and all other ranks retire at 57. Following the High
Court’s decision, the Centre issued a notification in August fixing the retirement age for all CAPF personnel
at 60.
Observing that discrimination in the age of retirement among members of the Central Armed Police
Forces (CAPFs) would hit morale, the court in January directed the Centre to fix a uniform age of retirement
for all personnel and officers of the CAPFs.

133. Based on the court’s verdict in the passage above, which of the following would be most correct:
(a) Court understands the reason for different age of retirement for different personnel.
(b) Court observed there is no discrimination in the age of retirement among members of the Central
Armed Police Forces (CAPFs)
(c) Court directed the Centre to fix a uniform age of retirement for all personnel and officers of the CAPFs
as these discrimination in the age of retirement among members of the Central Armed Police Forces
(CAPFs) would hit morale.
(d) Court directed the Centre to fix a uniform age of retirement for all personnel and officers of the CAPFs
as this discrimination in the age of retirement among members of the Central Armed Police Forces
(CAPFs) would be against right to equality and unpatriotic of country.

134. If the government still carries on with the different age of retirement for personnel based on the passage
above, which of the following would be most correct?
(a) Government would be following the courts order as Court understands the reason for different age of
retirement for different personnel.
(b) Government knows better about the work conditions of the its personnel not the court.
(c) Government would be going against the court continuing with the discrimination which would further hit
the morale of such personnel.
(d) This would make the army personnel understand that there is no point of fighting and dying for country
which is not even concerned about discrimination its army personnel are facing.

135. Following the High Court’s decision, the Centre issued a notification in August fixing the retirement age for
all CAPF personnel at 57. Based on the court’s verdict in the passage above, which of the following would
be most correct:
(a) Government would be following the courts order as Court as this would mean all the personnel have
common age of retirement.
(b) This would make the army personnel understand that there is no point of fighting and dying for country
which is not even concerned about discrimination its army personnel are facing.
(c) Government would be going against the court continuing with the discrimination which would further hit
the morale of such personnel.
(d) Government knows better about the work conditions of the its personnel not the court.

35
Section V: Quantitative Techniques

Directions for Questions 136 to 41: Study the table carefully and answer the given questions.

Data related number of candidates appeared and qualified in a competitive exam from 2 states during 5
years

State P State Q
Number of appeared Percentage of Number of Percentage of
Years candidates appeared candidates appeared appeared candidates
who qualified candidates who qualified
2006 450 60% - 30%
2007 600 43% - 45%
2008 - 60% 280 60%
2009 480 70% 550 50%
2010 380 - 400 -

Note: Few values are missing in the table (indicated by -). A candidate is expected to calculate the missing
value, if it is required to answer the given question on the basis of given data and information.

136. Out of the number of qualified candidates from State P in 2008, the respective ratio of male and female
candidates is 11 : 7. If the number of female qualified candidates from State P in 2008 is 126, what is the
number of appeared candidates (both male and female) from State P in 2008?
(a) 630 (b) 510 (c) 570 (d) 540

137. The number of appeared candidates from State Q increased by 100% from 2006 to 2007. If the total
number of qualified candidates from State Q in 2006 and 2007 together is 408, what is the number of
appeared candidates from State Q in 2006?
(a) 380 (b) 360 (c) 340 (d) 320

138. What is the difference between the number of qualified candidates from State P in 2006 and that in 2007?
(a) 12 (b) 22 (c) 14 (d) 24

139 If the average number of qualified candidates from State Q in 2008, 2009 and 2010 is 210, what is the
number of qualified candidates from State Q in 2010?
(a) 191 (b) 195 (c) 183 (d) 187

140. If the respective ratio between the number of qualified candidates from State P in 2009 and 2010 is 14:9,
what is the number of qualified candidates from State P in 2010?
(a) 252 (b) 207 (c) 216 (d) 234

36
Directions for Questions 141 to 145: Study the following graph carefully to answer the questions that
follow.

Speed (in Km/hr) of Three Trucks on Different Routes


on Different Days of the Week
60

50

40
Truck A
30
Truck B
20
Truck C
10

0
Mon Tues Wed Thur Fri Sat

Days

141. If the distance travelled by Truck A and Truck B on Saturday was the same, what was the respective ratio
of the time taken by Truck A to that taken by Truck B to travel that distance?
(a) 6:7 (b) 13:19 (c) 11:12 (d) None of these

142. If the distance travelled by Truck B on Thursday was 846 kms, what was the time taken by it to travel that
distance?
(a) 16 hrs 55 min 12 sec (b) 16 hrs 12 min
(c) 15 hrs 6 min 4 sec (e) None of these

143. What is the average speed of Truck A in metre/second for all the days together?
◌଼
(a) 180 (b) 13ଽ (c) 120 (d) None of these

144. Average speed of Truck C is approximately what per cent of average speed of Truck B over the days?
(a) 63 (b) 74 (c) 95 (d) 86

145. What is the per cent increase in speed of Truck C on Friday from the previous day?
(a) 15 (b) 28 (c) 20 (d) None of these

Directions for Questions 146 to 150: Read the information given in the passage and answer the given
questions.

There are ‘x’ number of students in a college. Each of them likes either one or more of the following
types of movies-Hollywood, Bollywood and Regional movies. The respective ratio of male and female
students in 9:7.
16% of the male students like only Hollywood movies. 22% like only Bollywood movies. 12% like only
Regional movies. 30% of the male students like only Hollywood and Bollywood movies. 10% like only
Bollywood and Regional movies and 6% like only Regional and Hollywood movies. The remaining 18 male
students like all the given types of movies.
14% of the female students like only Hollywood movies. 20% like only Bollywood movies. 3% like only
Regional movies. 26% of the female students like only Hollywood and Bollywood movies. 16% like only

37
Bollywood and Regional movies and 10% like only Regional and Hollywood movies. The remaining female
students like all the given types of movies.

146. What is the difference between the number of male students who like Bollywood movies and number of
female students who like the same?
(a) 69 (b) 59 (c) 63 (d) 65

147. Number of students (both male and female) who like all the given types of movies is approximately what
percent of the number of female students who like only one of the given types of movies?
(a) 12 (b) 18 (c) 32 (d) 27

148. Number of male students who like only two of the given types of movies is what percent more than the
number of female students who like only two of the given types of movies?
ଵ ଵ ଵଵ
(a) 11 (b) 16 (c) 9 (d) 13.74
ଶଵ ଶଵ

149. What is the respective ratio between number of female students who like Hollywood movies and number of
male students who like the same?
(a) 9:14 (b) 3:4 (c) 5:8 (d) 7:9

150. What is the value of ‘x’ (mentioned in the passage)?


(a) 960 (b) 800 (c) 640 (d) 720

New CLAT
LAW PREP MODULES PATTERN
Passages With Questions
Passages from NEWSPAPERS like
CLAT 2020
HINDU/Indian Express/HT
Passages From SC Judgements ENROLL
Passages from Bills/Acts/Articles
Passages from Online News Portals
NOW
www. lawpreptutorial.com

Avail your copy


JOIN LAW PREP TUTORIAL
ENQUIRY HELPLINE
076659 44999

38
1

Você também pode gostar